Difference between revisions of "2010 AIME I Problems/Problem 12"
(→Solution) |
(→Solution 2) |
||
(3 intermediate revisions by the same user not shown) | |||
Line 10: | Line 10: | ||
== Solution 2 == | == Solution 2 == | ||
− | Consider <math>{3,4,12}</math>. We could have any two of the three be together in the same set, and the third in the other set. Thus, we have <math>{3,4}, {3,12}, {4,12}</math>. We will try to 'place' numbers in either set such that we never have <math>a\cdot b = c</math>, until we reach a point where we MUST have <math>a\cdot b =c</math>. | + | Consider <math>\{3,4,12\}</math>. We could have any two of the three be together in the same set, and the third in the other set. Thus, we have <math>\{3,4\}, \{3,12\}, \{4,12\}</math>. We will try to 'place' numbers in either set such that we never have <math>a\cdot b = c</math>, until we reach a point where we MUST have <math>a\cdot b =c</math>. |
− | We begin with <math>{3,12}</math>. Notice that <math>a,b,c</math> do not have to be distinct, meaning we could have <math>3\cdot 3=9</math>. Thus <math>9</math> must be with <math>4</math>. Notice that no matter in which set <math>36</math> is placed, we will be forced to have <math>a\cdot b =c</math>, since <math>3*12=36</math> and <math>4*9=36</math>. | + | We begin with <math>\{3,12\}</math>. Notice that <math>a,b,c</math> do not have to be distinct, meaning we could have <math>3\cdot 3=9</math>. Thus <math>9</math> must be with <math>4</math>. Notice that no matter in which set <math>36</math> is placed, we will be forced to have <math>a\cdot b =c</math>, since <math>3*12=36</math> and <math>4*9=36</math>. |
− | We could have <math>{4,12}</math>. Similarly, <math>16</math> must be with <math>3</math>, and no matter to which set <math>48</math> is placed into, we will be forced to have <math>a \cdot b =c</math>. | + | We could have <math>\{4,12\}</math>. Similarly, <math>16</math> must be with <math>3</math>, and no matter to which set <math>48</math> is placed into, we will be forced to have <math>a \cdot b =c</math>. |
− | Now we have <math>{3,4}</math>. <math>9</math> must be with <math>12</math>. Then <math>81</math> must be with <math>{3,4}</math>. Since <math>27</math> can't be placed in the same set as <math>{3,4,81}</math>, <math>27</math> must go with <math>{9,12}</math>. But then no matter where <math>243</math> is placed we will have <math>a\cdot b =c</math>. | + | Now we have <math>\{3,4\}</math>. <math>9</math> must be with <math>12</math>. Then <math>81</math> must be with <math>\{3,4\}</math>. Since <math>27</math> can't be placed in the same set as <math>\{3,4,81\}</math>, <math>27</math> must go with <math>\{9,12\}</math>. But then no matter where <math>243</math> is placed we will have <math>a\cdot b =c</math>. |
Thus, <math>\boxed{243}</math> is the minimum <math>m</math>. | Thus, <math>\boxed{243}</math> is the minimum <math>m</math>. | ||
+ | |||
+ | ~skibbysiggy | ||
== Video Solution == | == Video Solution == |
Latest revision as of 21:13, 19 December 2024
Problem
Let be an integer and let . Find the smallest value of such that for every partition of into two subsets, at least one of the subsets contains integers , , and (not necessarily distinct) such that .
Note: a partition of is a pair of sets , such that , .
Solution 1
We claim that is the minimal value of . Let the two partitioned sets be and ; we will try to partition and such that the condition is not satisfied. Without loss of generality, we place in . Then must be placed in , so must be placed in , and must be placed in . Then cannot be placed in any set, so we know is less than or equal to .
For , we can partition into and , and in neither set are there values where (since and and ). Thus .
Solution 2
Consider . We could have any two of the three be together in the same set, and the third in the other set. Thus, we have . We will try to 'place' numbers in either set such that we never have , until we reach a point where we MUST have .
We begin with . Notice that do not have to be distinct, meaning we could have . Thus must be with . Notice that no matter in which set is placed, we will be forced to have , since and .
We could have . Similarly, must be with , and no matter to which set is placed into, we will be forced to have .
Now we have . must be with . Then must be with . Since can't be placed in the same set as , must go with . But then no matter where is placed we will have .
Thus, is the minimum .
~skibbysiggy
Video Solution
See Also
2010 AIME I (Problems • Answer Key • Resources) | ||
Preceded by Problem 11 |
Followed by Problem 13 | |
1 • 2 • 3 • 4 • 5 • 6 • 7 • 8 • 9 • 10 • 11 • 12 • 13 • 14 • 15 | ||
All AIME Problems and Solutions |
The problems on this page are copyrighted by the Mathematical Association of America's American Mathematics Competitions.